Difference between revisions of "2019 Mock AMC 10B Problems/Problem 16"
Excruciating (talk | contribs) |
|||
Line 11: | Line 11: | ||
Thus, <math> n = 14 + 1 = \boxed{(E) 15}.</math> | Thus, <math> n = 14 + 1 = \boxed{(E) 15}.</math> | ||
+ | |||
+ | Note: <math>g</math> does not need to be <math>\geq 1</math> because the inequality <math>a+b+c+d+e+f\leq n</math> is not strict. So this solution is incorrect. |
Latest revision as of 13:06, 11 August 2024
Solution by excruciating: For each of , they all need to be at least , so we have: or without the " need to be restriction." So now it's stars and bars:
which simplifies to which we can write as .
Expanding, we get .
Because we only have terms, is to be around the highest divisor. Thus, we must have as a divisor too. is one too. We have left is and , to make terms, so we have , and . Thus we have:
Thus,
Note: does not need to be because the inequality is not strict. So this solution is incorrect.